Difference between revisions of "2019 AMC 12B Problems/Problem 1"

(Redirected page to 2019 AMC 10B Problems/Problem 1)
(Tag: New redirect)
 
(2 intermediate revisions by one other user not shown)
Line 1: Line 1:
==Problem==
+
#REDIRECT[[2019_AMC_10B_Problems/Problem_1]]
 
 
==Solution==
 
 
 
Let the first jar's volume be <math>A</math> and the second's be <math>B</math>. It is given that <math>\frac{3}{4}A=\frac{5}{6}B</math>. We find that <math>\frac{B}{A}=\frac{3/4}{5/6}=\boxed{\frac{9}{10}}.</math>
 
 
 
We already know that this is the ratio of smaller to larger volume because it is less than <math>1.</math>
 
 
 
--mguempel
 
 
 
==See Also==
 
{{AMC12 box|year=2019|ab=B|num-b=12|num-a=14}}
 

Latest revision as of 13:13, 14 February 2019